LSAT and Law School Admissions Forum

Get expert LSAT preparation and law school admissions advice from PowerScore Test Preparation.

 Administrator
PowerScore Staff
  • PowerScore Staff
  • Posts: 8929
  • Joined: Feb 02, 2011
|
#30006
Please post below with any questions!
 ChicaRosa
  • Posts: 111
  • Joined: Aug 23, 2016
|
#30263
I was stuck between B and E and ended up choosing the wrong answer instead. Why is B wrong while E is correct?

I know in weakening questions you show an alternative explanations for the conclusion and if the reasoning is cause and effect you prove that the effect doesn't happen when the cause is started or show that the effect is the cause.

I thought B was correct because even if the substances don't cause effects that relate with Alzheimer's disease but can be harmful in other ways to the persons health and I thought it would weaken the conclusion about reducing Alzheimer's disease by having a diet of large amounts of B vitamins and folic acid which converts homocysteine into substances that don't relate with Alzheimer's disease.

Is E correct because it's showing the effect (Alzheimer's disease) being the cause of homocysteine?

Thanks!
 David Boyle
PowerScore Staff
  • PowerScore Staff
  • Posts: 836
  • Joined: Jun 07, 2013
|
#30336
ChicaRosa wrote:I was stuck between B and E and ended up choosing the wrong answer instead. Why is B wrong while E is correct?

I know in weakening questions you show an alternative explanations for the conclusion and if the reasoning is cause and effect you prove that the effect doesn't happen when the cause is started or show that the effect is the cause.

I thought B was correct because even if the substances don't cause effects that relate with Alzheimer's disease but can be harmful in other ways to the persons health and I thought it would weaken the conclusion about reducing Alzheimer's disease by having a diet of large amounts of B vitamins and folic acid which converts homocysteine into substances that don't relate with Alzheimer's disease.

Is E correct because it's showing the effect (Alzheimer's disease) being the cause of homocysteine?

Thanks!

Hello,

As for "I know in weakening questions you show an alternative explanations for the conclusion and if the reasoning is cause and effect you prove that the effect doesn't happen when the cause is started or show that the effect is the cause", there may be other ways to weaken as well, e.g., the effect without the cause; or an alternative cause; or data problems.
Answer B is incorrect because we don't really care in this problem about bad effects besides Alzheimer's.
As for "Is E correct because it's showing the effect (Alzheimer's disease) being the cause of homocysteine?", yes.

David
 Mb1995
  • Posts: 1
  • Joined: Nov 07, 2017
|
#41340
Hi, I don’t really understand why C is incorrect. Can you explain why E is better than C?

I appreciate the help!
 Eric Ockert
PowerScore Staff
  • PowerScore Staff
  • Posts: 164
  • Joined: Sep 28, 2011
|
#41459
Hi!

Answer choice (C) addresses taking B vitamins and folic acid in the form of vitamin-mineral supplements. However, the stimulus speaks only of "including in one's diet large amounts" of these two substances. Since the stimulus is not directly indicating this increase is due to vitamin-mineral supplements, answer choice (C) is a Shell Game answer that would have no effect on the stimulus.

Answer choice (E), on the other hand, indicates that homocysteine is an effect of Alzheimer's, not the cause (which is what the author implies in the argument). Anytime an answer choice suggests that the author's causal argument might be reversed, that will weaken that causal argument.

So, it isn't so much that answer choice (E) is better than answer choice (C). Answer choice (C) really does not have any impact on the argument, while (E) is the only answer that does weaken the argument.

Hope that helps!
 EmiliaGrace
  • Posts: 5
  • Joined: Jun 01, 2018
|
#46127
Hi,
I understand now why E is the correct answer. I had originally choosen A because I was confused by the wording. Is answer A already in the stimulus and so it would not weaken the argument?
 James Finch
PowerScore Staff
  • PowerScore Staff
  • Posts: 943
  • Joined: Sep 06, 2017
|
#46130
Hi Emilia,

A big red flag with answer choice (A) is the word "many:" because the word is so vague, it will likely have only a small impact, if any, on the likelihood that the stimulus's conclusion is true. Moreover, the conclusion in the stimulus is concerned with homocysteine as one cause of Alzheimer's, and whether reducing levels of it in the blood would reduce likelihood of contracting Alzheimer's. So just because many Alzheimer's patients have normal homocysteine levels, and presumably had them before they contracted Alzheimer's, doesn't mean that lowering homocysteine levels for those who have higher-than-normal levels will make it less likely for people in that group to eventually contract the disease.

Hope this clears things up!
 HowardQ
  • Posts: 32
  • Joined: Jun 25, 2018
|
#50329
Hi,

It's interesting to see how people find each wrong answer appealing, and I have some question about D. D provides an alternative cause to the relationship, is it wrong because the causal relationship was not explicitly stated so that genetics is not the only cause of the disease? While E indicates a reversed relationship of the implied partial causality between H and A disease?

Thanks,
 LSAT2018
  • Posts: 242
  • Joined: Jan 10, 2018
|
#59118
Although I chose answer (E), I too, had a difficult time eliminating answer (D) like the previous poster. Genetics seemed like an alternate cause for Alzheimer's disease. Is (E) the correct answer because it specifically refers to homocysteine in the blood as the cause (when referring to including B vitamins and folic acid in one's diet)? So even if genetics is an alternate cause, it doesn't seem to affect the B vitamins and folic acid part?
 Brook Miscoski
PowerScore Staff
  • PowerScore Staff
  • Posts: 418
  • Joined: Sep 13, 2018
|
#59512
Howard and 2018,

Reading through the stimulus, it is based on causal/statistical reasoning. The fact that homocysteine levels are elevated in Alzheimer's patients does not establish a causal relationship, and that should be attacked when weakening the conclusion. Answer choice (E) does that by reversing the proposed cause and effect, and that's a great reason for choosing (E).

The problem with (D) is that showing an additional cause does not eliminate homocysteine as a cause. Homocysteine could be an independent factor, or genetics could cause homocystein which in turn causes Alzheimer's.

The LSAT test writers have by now figured out that people are looking for alternative causes, so they will try to trip you up. You should still be looking for all the standard attacks on causal reasoning, but don't despair if you have more than one option. Just think about which choice is best at ruining the causal relationship in the stimulus. It's not enough to change it slightly or make it more complex if you have the chance to ruin it.

Get the most out of your LSAT Prep Plus subscription.

Analyze and track your performance with our Testing and Analytics Package.